PT74.S4.Q24 - The makers of Activite, a natural dietary

zackrynholdzackrynhold Free Trial Member
edited July 2018 in Logical Reasoning 5 karma

Why is the answer to this question, D? The passage says that they sold the product for free, so how are we to know whether this includes shipping and handling? Is B not a more appropriate answer?

Explanation: https://7sage.com/lsat_explanations/lsat-74-section-4-question-24/

Admin note: edited title and added video explanation link

Comments

  • eRetakereRetaker Free Trial Member
    2043 karma

    Hi @zackrynhold, B is a classic trap answer which would be good practice for you to recognize for future assumption family questions. You have to pay special attention to what the conclusion is saying. The conclusion simply claims that "Activite must be effective." It does not claim that there aren't less expensive supplements on the market. All the conclusion says is that Activite is effective. Think about it this way. Does the rebuttal in B actually say that Activite is not effective?

    D is correct answer and you are going to have to interpret these answers a bit more liberally. When they say that they offer a month's supply of Activite for free, that just means the expense of the drugs themselves. The handling fee would be considered a separate revenue that covers the cost of the "free" supply (making the company seem a bit more shady), thus weakening the claim. Once you encounter more LR questions involving costs, you'll notice that a lot of them are not overly inclusive of all the expenses that can involved. Your judgement will get better with practice. I personally seen this exact type of question several times in earlier PTs that tests you on noticing the distinctions similar to answer choices B and D.

  • jhbm_nycjhbm_nyc Alum Member
    edited July 2018 568 karma

    This is a really hard weaken Q. The argument goes like this:
    P: Activite offers a free month's supply to new customers.
    P: If Activite weren't effective, this free offer would go against the company's interest.
    C: Activite must be effective.

    The argument assumes that the just because the free offer doesn't go against the company's interest, Activite must be effective. (D) attacks this assumption by claiming that the company charges a handling fee that's "considerably more" than the actual shipping cost. So Activite could totally be ineffective and the company is just using the free offer to get new customer's money.

    As @eRetaker explained above, we don't care about other products (B). The fact that there are other products that are "just as effective" has no bearing on whether Activite is actually effective. Maybe they're all super effective or equally ineffective. Test writers often use irrelevant or unhelpful comparisons to write wrong answer choices.

  • zackrynholdzackrynhold Free Trial Member
    5 karma

    Thanks guys

Sign In or Register to comment.